Retirement Planning Review Questions

¡Supera tus tareas y exámenes ahora con Quizwiz!

Which of the following is/are elements of an effective waiver for a preretirement survivor annuity? 1. The waiver must be signed within six months of death. 2. The waiver must be signed by a plan participant. 3. The waiver must be signed by the nonparticipant spouse and notarized or signed by a plan official. A. 3 only B. 1 and 2 C. 2 and 3 D. 1, 2, and 3

Answer: A Only the nonparticipant spouse must sign the waiver.

Devon was awarded 1,000 shares of restricted stock of B Corp at a time when the stock price was $14. Assume Devon properly makes an 83b election at the date of the award. The stock vests 2 years later at a price of $12 and Devon sells it then. What are Devon's tax consequences in the year of sale? A. Devon has W-2 income of $12,000 B. Devon has a long-term capital loss of $2,000 C.Devon has W-2 income of $14,000 D. Devon has a $12,000 long-term capital gain.

Answer: B In the ear of sale, Devon will have a long-term capital loss of $2,000 ($14,000 - $12,000) because his right to the stock vested. Losses are permitted when 83b is elected after the right to the stock has vested.

Basil Leaf, Inc. sponsors a 25% money purchase pension plan for its eligible employees. Brent earns $210,000, Al earns $60,000, Rosie earns $275,000, and Tim, who is ineligible, earns $27,000. What is Basil Leaf's required deductible contribution for the year? All employees are under the age 50. A. $117,000 B. $120,000 C. $133,750 D. $136,250

Answer: C $210,000 + $60,000 + $265,000 (Max Covered Compensation Limit, NOT $275,000) X .25 = $133,750

What is the maximum number of employees that a company with a health plan can have and not be subject to the COBRA rules? A. 10 B. 15 C. 19 D. 20

Answer: C A company with fewer than 20 employees is not subject to COBRA even if it has a health plan.

For 2016, what is the maximum amount that can be contributed to a SEP? A. $18,000 B. $24,000 C. $53,000 D. $59,000

Answer: C For 2016, the maximum contribution for an individual to a SEP is the lesser of: 25% of compensation (compensation maximum is $265,000), or $53,000. Therefore, the maximum contribution to a SEP for 2016 is $53,000 ($265,000 maximum compensation X 25% , limited to $53,000). SEPs do not permit catch up contributions since they are 100% employer funded.

Rhianna received 1,000 NQSOs with an exercise price of $25 per share when the stock was $25 on the market. Two years from the date of grant, Rhianna exercises when the stock price is $102. Rhianna: A. Has W-2 income of $25,000 B. Has a AMT adjustment of $77,000 C. Has W-2 income of $77,000 D. Has an AMT adjustment of $25,000

Answer: C Rhianna will have W-2 income of the difference between the market price and the exercise price. She will not have an AMT adjustment for the exercise of an NQSO.

Laura has self-employment income of $87,500 for the year. How much is her self-employment tax? A. $10,747 B. $11,637 C. $12,363 D. $13,387

Answer: C $87,500 X .9235 X .153 = $12,363

Juliet is married top Jack and they have one child Angela, age 14, who is in the 6th grade. Angela is a difficult child and she is cared for in the afternoon by the Sisters of Reformation, a group of Catholic nuns. Juliet pay $6,000 per year for the child care. Juliet's company has a dependent care assistance program. If Juliet makes the maximum use of the dependent care assistance program, how much can she exclude from her income if she files a joint return with Jack? A. $0 B. $2,500 C. $5,000 D. $6,000

Answer: A Angela is over 13 years old and, therefore, does not qualify for the dependent care assistance program.

JMG Company has three employees: Julia, Maria, and Gary. Their compensations are $50,000, $150,000, and $200,000 respectively. JMG is considering establishing a straight 10 percent profit sharing plan or an integrated profit sharing plan using a 10 percent contribution for base compensation and 15.7 percent for excess compensation. Which of the following statement is correct ? A. If the integrated plan is selected, then the total contribution for all employees is $46,441. B. The effect of the integrated plan results in an increase in Maria's contribution of $4,645. C. If the integrated plan is selected, the base contribution for all employees is $40,000. D. If the integrated plan is selected, Gary's total contribution is $30,031

Answer: A If JMG selected the 10 percent profit sharing plan, the amount for the employer contributions for would $5,000 for Julia, $15,000 for Maria, and $20,000 for Gary. Alternatively, if JMG established an Integrated Plan using a 10 percent base contribution and a 15.7 percent excess contribution, more benefit could be allocated to Maria and Gary. Using an integrated plan, Maria would receive a $16,796 contribution ($1,796 more than with a straight 10 percent profit sharing plan) and Gary would receive an $24,645 contribution ($4,645 more than with a straight 10 percent profit sharing plan)

On April 30, Ava, age 42, received a distribution from her qualified plan of $150,000. She had an adjusted basis in the plan of $500,000 and the fair market value of the account as of April 30 was $625,000. Calculate the taxable amount of the distribution. and any applicable penalty. A. $30,000 taxable, $3,000 tax penalty B. $30,000 taxable, $0 tax penalty C. $120,000 taxable, $12,000 tax penalty D. $150,000 taxable, $15,000 tax penalty

Answer: A Pro Rata taxed amount = $125,000 / $625,000 X $150,000 = $30,000 Tax Penalty = $30,000 X .10 = $3,000

Which of the following statements concerning tax considerations of non-qualified retirement plans is (are) correct? 1. Under IRS regulations an amount becomes currently taxable to an executive even before is actually received if it has been "constructively received." 2. Distributions from non-qualifiedk retirement plans are generally subject to payroll taxes. A. 1 only B. 2 only C. 1 and 2 D. Neither 1 nor 2

Answer: A Statement 2 is incorrect because payroll taxes are due on deferred compensation at the time the compensation is earned and deferred, not at the date of distribution. Statement 1 is a correct statement.

Caden sells stock several years after he received it as a distribution from a qualified stock bonus plan. Qhen the stock was distributed, he had a net unrealized appreciation of $7,500. Caden also had ordinary income from the distribution of $29,000. The fair market value of the stock and the sales price at the time of the sale was $81,000. How much of the sales price will be subject to long-term capital gain treatment? A. $7,500 B. $44,500 C. $52,000 D. $73,500

Answer: C Sale Price - Adjusted basis. $81,000 - $29,000 = $52,000 long term capital gain.

Which of the following is true regarding employer contributions to secular trusts for employee-participants of a non-qualified deferred compensation agreement? 1. Participants have security against an employer's unwillingness to pay at termination. 2. Participants have security against an employer's bankruptcy. 3. Secular trust provide tax deferral for employees until distribution. 4. Secular trusts provide employers with a current income tax deduction. A. 3 only B. 1 and 2 C. 1, 2, and 4. D. 1,2,3 and 4

Answer: C Secular trusts are similar to rabbi trusts except that participants do not have a substantial risk of forfeiture and thus, do not provide the employee with tax deferral. Secular trusts provide the employer with a current income tax deduction for contributions. Secular trusts protect the participant from employer unwillingness to pay because they are funded, and they protect from bankruptcy because there is no risk of forfeiture.

Johnson Brothers Co. has a noncontributory qualified profit sharing plan with 310 employees in total, 180 are nonexcluable (40 HC and 140 NHC). The plan covers 72 NHC and 29 HC. The NHC receive an average of 4.5% benefit and the HC receives 6.5%. Which of the following statement is (are) correct? 1. The plan meets the ratio percentage test. 2. The plan fails the average benefits test. 3. The plan must and does meet the Actual Deferral Percentage (ADP) test. A. 1 only B. 2 only C. Both 1 and 2 D. 1, 2, and 3

Answer: C The plan does not have to meet the ADP test because it is a noncontributory plan. The plan meets the ratio percentage test and fails the average benefits test.

Coldstone Company allows a 25% discount to all non-officer employees. Officers are allowed a 30% discount on company products. Coldstone's gross profit percent is 35%. Which of the following is true? A. An officer who takes a 30% discount must include the extra 5% (30% - 25%) in his gross income. B. Any discounts taken by any employee is includable in the employee's gross income because the plan is discriminatory. C. All discounts taken by officers (30%) are includable in their gross income because the plan is discriminatory. D. Non of the discounts taken by any employee are includable in their gross income because the discount, in all cases, is less than the company's gross profit percentage.

Answer: C The plan is discriminatory to non-highly compensated employees; therefore, all discounts actually taken by officers are includable in the officers' income, not just the excess of what is available to the non officers. Any discount taken by a non-officer would be excluded from the employees' gross income.

On July 31, 2001, B Corp sold 1,000 shares of its stock to Dowe, an employee, for $12 a share. At the time of the sale, B stock was trading for $30 per share. The stock was to best in 4 years on July 31, 2005, the B stock was trading for $125 per share. Dowe sold the stock in 2006 for $125 per share. Assuming no special elections, how much must Dowe include in income and in what year? A. 2001, $18000 B. 2001 - 2003, $18,000 ratably C. 2005, $113,000 D. 2006, $113,000

Answer: C When the substantial risk of forfeiture expires on July 31, 2005, Dowe has income of $125,000 less the basis of $12,000 or $113,000.

What is the first year in which a single taxpayer, age 58 in 2001, could receive a qualified distribution from a Roth IRA, if he made a $4,000 contribution to the Roth IRA on April 1, 2002, for the tax year 2001? A. 2003 B. 2004 C. 2005 D. 2006

Answer: D A qualified distribution can only occur after a five-year period has occurred and is made on or after the date on which the owner attains age 59.5, made to a beneficiary or the estate of the owner on or after the date of the owner's death, attributable to the owner's being disabled, or for a first-time home purchase. The five year period begins at the beginning of the taxable year of the initial contribution to a Roth IRA. The five year period ends on the last day of the individual's fifth consecutive taxable year beginning with the taxable year described in the preceding sentence. Therefore, the first year in which a qualified distribution could occur is 2006.

Which of the following pension plans would allocate a higher percentage of the plan's current costs to a certain class or group of eligible employees? 1. Defined benefit pension plan 2. Target benefit pension plan 3. Money purchase pension plan with permitted disparity A. 1 only B. 1 and 2 C. 2 and 3 D. 1,2, and 3.

Answer: D All of the pension plans listed would allocate a higher percentage of the plans' current costs to a certain class or group of eligible employees. A defined benefit pension plan and a target benefit pension plan would allocate a higher percentage of the plan's current costs to the older participants. A money purchase pension plan with permitted disparity would allocate a higher percentage of the plan's current cost to those participants whose earnings are in excess of the Social Security wage base (or integration level if different)

Which of the following plans permit employers to match employee elective deferral contributions or make nonelective contributions? 1. 457b 2. 401k 3. 403b A. 2 only B. 3 only C. 2 and 3 D. 1, 2, and 3

Answer: D All three plans permit employer matching and nonelective contributions. The 457 employer contribution goes against the $18,000 limit, whereas employer contributions do not go against the $18,000 limit for 401k and 403b plans.

Rex, a married 29 year old, deferred 10% of his salary, or $10,000, into a 401k plan sponsored by his employer this year. His wife was unemployed all year and did not receive unemployment compensation. Assuming Rex has no other income, what is the maximum contribution Rex's wife can make to her Roth IRA for this year? A. $0 B. $1,000 C. $4,500 D. $5,500

Answer: D Rex's wife can make a $5,500 Roth IRA contribution, the maximum for this year, because Rex's AGI of $100,000 is below the phase out limit of $184,000. Even though she does not have any earned income of her own, she can use Rex;s earnings to qualify for the contribution.

Which of the following is not a qualified retirement plan? A. ESOP B. 401(k) Plan C. 403(b) Plan D. Target Benefit Plan

Answer: C 403(b) is a tax-advantaged plan, not a qualified plan. All of the others are qualified plans.

A distress termination of a qualified retirement plan occurs when: 1. The PBGC initiates a termination because the plan was determined to be unable to pay benefits from the plan. 2. An employer is in financial difficulty and is unable to continue with the plan financially. Generally this occurs when the company has filed for bankruptcy, either Chapter 7 liquidation or Chapter 11 reorganization. 3. The employer has sufficient assets to pay all benefits bested at the time, but is distressed about it. 4. When the PBGC notifies the employer that it wishes to change the plan due to the increasing unfunded risk. A. 2 only B. 1 and 2 C. 1, 2, and 3. D. 1, 2, and 4.

Answer: A Statement 2 is the definition of a distress termination. Statement 3 is standard termination. Statement 1 describes involuntary termination. Statement 4 is simply false.

Dimitri operates Downtown Discount Pharmacy, a sole proprietorship. Downtown Discount Pharmacy sponsors a profit sharing plan. Dimitir had net income $205,000 and paid self0employment taxes of $20,184 for the year. If Dimitri makes a 15% of salary contribution on behalf of all of his employees to the profit sharing plan, how much is the contribution to the profit sharing plan on behalf of Dimitri? A. $22,820 B. $25,416 C. $26,250 D. $30,750

Answer: B $205,000 - $10,092 (1/2 SE Tax) = $194,908 $194,908 X .1304 (.15/1.15) = $25,416

Andrew, age 53, had the following items of income: Investment returns as a limited partner in a partnership of $1,200 Unemployment compensation of $350 Income from a law practice of $600 Deferred compensation from a former employer of $14,000, not constructively received this year. Alimony of $750 Wage of $1,000 What is the maximum contribution Andrew can make to an IRA for this year? A. $1,750 B. $2,350 C. $2,700 D. $5,000

Answer: B Andrew is limited to making an IRA contribution equal to the lesser of $6,500 (2016) (including the catch-up) or his earned income for the year. Andrew's earned income includes his law practice income, alimony, and wage which total $2,350. t

Marni received 1,000 SARs at $22, the current trading price of Clippers, Inc., her employer. If Marni exercises the SARs three years after the grant and Clipper's stock is $34 per share, which of the following statement is true? A. Marni will have an adjusted basis of $22,000 in the Clippers, Inc. stock. B. Marni will have W-2 income equal to $12,000. C. Marni will have long-term capital gain of $12,000. D. Marni will have ordinary income equal to $22,000

Answer: B At the exercise of a SAR, the employee receives the difference between the fair market value and the exercise price as W-2 income. Thus, Marni has W-2 income equal to $12,000 ($34 - $22) X 1,000

Caleb, the 100 percent owner of Caleb's Car Wash (a sole proprietorship), would like to establish a profit sharing plan. Caleb's Car Wash's tax year ends July 31st to coincide with the school year. What is the latest day Caleb can establish and contribute to the plan? A. Caleb must establish and contribute to the plan by December 31st of the year in which he would like to establish the plan. B. Caleb must establish the plan by July 31st of the year in which he would like to have the plan and contribute by May 15th of the following year assuming he filed the appropriate extensions. C. Caleb must establish the plan by July 31st of the year in which he would like establish the plan and contribute by December 31st. D. Caleb must establish the plan by December 31st of the year in which he would like to establish the plan and contribute to the plan by April 15th of the following year.

Answer: B Caleb must establish the plan by July 31st of the year in which he would like to have the plan and contribute funds by May 15th (9.5 months after year end) the following year assuming he filed all of the appropriate extensions.

Sawyer Outboards, a C Corporation, has four employees. The company sponsors a proft sharing plan and contributed10 percent of employee compensation for the current year to the plan. The company has the following employee information. There is no state income tax and federal withholding is 15% of gross pay. Davis = $50,000 Salary, $5,000 Profit Sharing Contribution Lou = $40,000 Salary, $4,000 Profit Sharing Contribution Allan = $30,000 Salary, $3,000 Profit Sharing Contribution Katie = $20,000 Salary, $2,000 Profit Sharing Contribution Which of the following statements are true? A. Sawyer Outboards will not be able to take a deduction for the contribution to the profit sharing plan. B. After payroll taxes and withholding, Davis will only receive $38,675 in take home salary. C. Sawyer Outboards will pay total payroll taxes of $11,781 D. Lou must include the $4,000 contribution to the profit sharing plan made on her behalf in her gross income for the current year.

Answer: B Davis will be responsible for payroll taxes on the $50,000 he receives in salary. His payroll will be reduced by 7.65 of $50,000 plus $7,500 withholding, this he will receive $38,675. Sawyer Outboards may deduct the contribution to the profit sharing plan. Sawyer Outboards will only pay payroll taxes of $10,710. Amounts contribute to the profit sharing plan are not subject to payroll taxes. Contributions to profit sharing plans are not included in the employees gross income tax at the date of distribution.

Ethan, age 60, is a participant in the stock bonus plan of Galloway, Inc., a closely held corporation. Ethan received contributions in shares to the stock bonus plan and Galloway, Inc. took income tax deductions as follows: 2012 = 100 Shares @ $10 2013 = 125 Shares @ $12 2014 = 150 Shares @ $13 2015 = 200 Shares @ $15 2016 = 400 Shares @ $18 Ethan terminates employment and takes a distribution from the plan of 975 shares of Galloway, Inc., having a fair value of $19,500. What are Ethan's tax consequences? A. There are no immediate tax consequences because he has not sold the stock. B. Ethan has an ordinary income of $14,650 at distribution C. Ethan has net unrealized appreciation of $19,500 at distribution. D. Ethan has ordinary income of $19,500 at distribution

Answer: B Ethan's ordinary income is exactly equal to to Galloway's deduction at the time of contribution, $14,650. Ethan's net unrealized appreciation is $4,850 ($19,500 - $14,650) and will be taxed as long-term capital gains when the stock is sold.

Which of the following is false regarding a deferred compensation plan that is funded using a rabbi trust? 1. Participants have security against the employer's unwillingness to pay. 2. Rabbi trusts provide the participant with security against employer bankruptcy 3. Rabbi trusts provide tax deferral for participants. 4. Rabbi trusts provide the employer with a current tax deduction. A. 3 only B. 1 and 2 C. 1, 2, and 4 D. 1, 2, 3, and 4.

Answer: B Rabbi trusts do not provide security against employer bankruptcy or a current tax deduction for the employer.

A SEP is not a qualified plan and is not subject to all of the qualified plan rules; however, it is subject to many of the same rules. Which of the following are true statements? 1. SEPs and qualified plans have the same funding deadlines. 2. The contribution limit for SEPs and qualified plans (defined contribution) is $53,000 for the year 2016. 3. SEPs and qualified plans have the same ERISA protection from creditors 4. SEPs and qualified plans have different nondiscriminatory and top-heavy rules. A. 1 only B. 1 and 2 C. 2 and 4 D. 1, 2, 3, and 4

Answer: B SEPs and qualified plans can be funded as late as the due date of the return plus extensions. The maximum contribution for an individual to a SEP is $53,000 for 2016 ($265,000 maximum compensation X 25%, limited to $53,000). Thus, Statements 1 and 2 are correct. Qualified plans are protected under ERISA. IRAs and SEPs do not share this protection. Both types of plans have the same nondiscriminatory and top-heavy rules.

Which of the following statements is/are correct regarding SEP contributions made by an employer? 1. Contributions are subject to FICA and FUTA 2. Contributions are currently excludable from employee-participant's gross income. 3. Contributions are capped at $18,000 for 2016. A. 1 only B. 2 only. C. 1 and 2 D. 1, 2 and 3.

Answer: B Statement 2 is the only correct response. Statements 1 and 3 are incorrect. Employer contributions to a SEP are not subject to FICA and FUTA. The 401k elective deferral limit and the SARSEP deductible limits are $18,000 for 2016. The SEP limit is 25% of covered compensation up to $53,000 for 2016.

Which of the following statements is/are correct regarding TSAs and 457 deferred compensation plans? 1. Both plans require contracts between an employer and an employee. 2. Participation in either a TSA or a 457 plan will cause an individual to be considered an "active participant" for purposes of phasing out the deductibility of traditional IRA contributions. 3. Both plans allow 10-year forward averaging tax treatment for lump-sum distributions 4. Both plans must meet minimum distribution requirements that apply to qualified plans. A. 1 only B. 1 and 4 C. 2, 3, an 4 D. 1, 2 and 4.

Answer: B Statements 1 and 4 are correct. Statement 2 is incorrect because a 457 plan is a deferred compensation arrangement that will not cause a participant to be considered an "active participant". Statement 2 is incorrect because 10-year forward averaging is not permitted form either plan.

Tori receives ISOs with an exercise price equal to the FMV at the date of the grant of $22. Tori exercises these options 3 years from the date of the grant when the FMV of the stock is $30. Tori Then sells the stock 3 years after exercising for $35. Which of the following statement is (are) true? 1. At the date of grant, Tori will have ordinary income equal to $22. 2. At the ate of exercise, Tori will have W-2 income of $8. 3. At the date of sale, Tori will have long0term capital gain of $13. 4. Tori's employer will not have a tax deduction related to the grant, exercise or sale of this ISO by Tori. A. 3 only. B. 3 and 4. C. 2, 3, and 4. D. 1,2, and 4.

Answer: B Statements 3 and 4 are correct. Since Tori held the underlying security 2 years from grant and one year from exercise before its sale, Tori will receive long0term capital gain treatment for the appreciation, and her employer will not have a deductible expense related to the ISO.

Sophia, who turned 81 in December 2015, takes a lump-sum cash distribution of $600,000 from her former employer's profit sharing plan during 2016. Sophia had worked for the employer since 1972. Which of the following taxation options are available for Sophia in the year of this distribution? 1. Ten-year forward averaging. 2. Pre-74 capital gain treatment 3. Net unrealized appreciation 4. Five-year forward averaging A. 1 and 4 B. 1 and 2 C. 1, 2, and 3. D. 2, 3, and 4.

Answer: B Ten year averaging and pre-74 capital gain treatment are available because sophia was born in 1934 and she is taking a lump sum distribution. Net unrealized appreciation is not available because SOphia has taken a cash distribution. Five-year averaging is no longer an option for lump-sum distributions.

Which of the following statements regarding 403b plans is true? 1. Assets within a 403b plan may be invested in individual securities 2. A 403b plan usually provides a 3-7 year graduated vesting program 3. A 403b plan must pass the ACP test if it is an ERISA plan. 4. In certain situations, a participant of a 403b plan can defer an additional $9,000 as a catch up to the 403b plan. A. 4 only B. 1 and 2 C. 3 and 4 D. 2,3, and 4

Answer: C 403b plan assets cannot be invested in individual securities, and deferrals to 403b accounts are always 100% vested. Statements 3 and 4 are true. Remember that 403b can have both the catch up for age 50 and over ($6,000) and 15 year rule ($3,000)

Elizabeth = $50,000 (Salary), 5% (Ownership-2016), 95% (Ownership-2015) Carol = $50,000 (Salary), 95% (Ownership-2016), 5% (Ownership-2015) David = $150,000 (Salary), 0% (Ownership-2016), 0% (Ownership-2015) Which of the three employees are considered highly compensated based on the qualified plan rules? A. Elizabeth B. Carol and David C. Elizabeth, Carol and David D. David

Answer: C All three individuals are considered highly compensated for the current year.

Dita began taking RMDs from her profit sharing plan 7 years ago. In 2016, Dita died after suffering a heart attack. She had not named a beneficiary of her profit sharing plan. Which of the following statements is false? A. Dita's estate may take a full distribution of the profit sharing plan's assets in the year of her death. B. In the year of Dita's death the RMD will equal the minimum required distribution had Dita not died. C. Dita's estate must take a distribution of the profit sharing plan account balance by the end of 2021. D. The RMD for 2017 will be calculated utlizing the factor according to Dita's age reduced by one.

Answer: C Because Dita had already begun required minimum distributions, the five-year requirement does not apply, but her estate must continue taking distributions over Dita's remaining life expectancy reduce by one year. All of the other options are true statements.

Marshall has a qualified plan with an account balance of $2,000,000. In which of the following circumstances would a third party be able to alienate the assets within Marshall's qualified plan? 1. A QDRO in favor of a former spouse 2. A federal tax levy. 3. Creditors in a personal bankruptcy A 3 only B. 1 and 3 C. 1 and 2 D. 1, 2, and 3

Answer: C Because a qualified plan is designed to provide individuals with income at their retirement, ERISA provides an anti-alienation protection over all assets within a qualified plan. This anti-alienation protection prohibits the plan assets from being assigned, garnished, levied, or subject to bankruptcy proceedings while the assets remain in the plan so that the individual has income at their retirement. Qualified plan assets are not protected from alienation due to a qualified domestic relations order, a federal tax levy, or from a judgment or settlement rendered upon an individual for criminal act involving the otherwise protected qualified plan.

Which of the following statements regarding determination letters for qualified plans is true? A. When a qualified plan is created, the plan sponsor must request a determination letter from the IRS. B. An employer who adopts a prototype plan must request a determination letter from the IRS. C. If a qualified plan is amended, the plan sponsor must request a determination letter from the Department of Labor. D. A qualified plan which receives a favorable determination letter from the IRS may still be disqualified at a later date.

Answer: D Determination letters are issued by the IRS at the request of the plan sponsor. The plan sponsor is not required to request a determination letter. Even if the the determination letter is requested and approved, the IRS may still disqualify at a later date.

Ella, age 70 on February 2, YR4, had the following account balances in a qualified retirement plan: 12/31/YR1 = $300,000 12/31/YR2 = $350,000 12/31/YR3 = $500,000 12/31/YR4 = $478,000 12/31/YR5 = $519,000 12/31/YR6 = $600,000 Assuming that Ella is retired and has never taken a distribution prior to YR5, what is the total amount of minimum distribution in YR5? Life expectancy factors according to the uniform life table are 27.4 for a 70 year old and 26.5 for a 71 year old. A. $18,037 B. $18,248 C. $35,597 D. $36,286

Answer: D For YR4, look back to YR3: $500,000 / 27.4 = $18,248 For YR5, look back to YR4: $478,000 / 26.5 = $18,038 $18,248 + $18,038 = $36,286 Question asks what is the TOTAL amount of RMD. Since she can claims first RMD in April of YR5, you would need to include both YR4 and YR5 in YR5.

Which of the following is true regarding QDROs? A. The court determines how the retirement plan will satisfy the QDRO (i.e., split accounts, separate interest). B. In order for a QDRO to be valid, the order must be filed on Form 2932-QDRO provided by ERISA C. All QDRO distributions are charged a 10% early withdrawal penalty. D. A QDRO distribution is not considered a taxable distribution if the distribution is deposited in the recipient's IRA or qualified plan.

Answer: D The plan document, not the court, determines how the QDRO will be satisfied. No particular form is required for a QDRO, although some specific information is required. Form 2923-QDRO is not a real form. QDRO distributions may be subject to the 10% early withdrawal penalty if the distribution is not deposited into the recipient's IRA or qualified plan.

ABC Corp. provides its employees with discounts on the flat panel televisions they manufacture. The discounts were established using a length of service and employee status methodology. 1 - 5 years = 10% employee discount, 20% officer discount >5 - 10 years = 15% employee discount, 30% officer discount >10 years = 20% employee discount, 40% officer discount The gross profit percentage for ABC is 40% James is an officer-employee who has been with ABC Corp. for 13 years. For Christmas this year, James bough a 56-inch flat panel television that retails for $8,800 and he received a discount appropriate to the schedule discounts listed above. For this year, how much, if any, does James have to include in gross income as a result of this transaction? A. $0 B. $1,760 C. $2,640 D. $3,520

Answer: D The plan is clearly discriminatory; therefore, James, who is an officer, must include the entire discount of $3,520 ($8,800 X .40)


Conjuntos de estudio relacionados

Unit 3: Geometry in Engineering, Architecture, and Fine Arts, Part 2

View Set

Behavior of Domestic Animals Test 4

View Set

Ch.7 P2 Linear Regression with Categorical Variables

View Set

Nervous System, Sensory Organs & Action Potentials

View Set

Give combining forms for the following meanings:

View Set

Chapter 18 - The Circulatory System: Blood

View Set